Problems proving $e^{n+1} gt (1+n)^2$ [closed]











up vote
0
down vote

favorite












I'm trying to prove by induction, but I'm getting stuck in the middle.



I'd use some help.










share|cite|improve this question













closed as off-topic by amWhy, José Carlos Santos, Cesareo, Chinnapparaj R, user302797 Nov 23 at 3:09


This question appears to be off-topic. The users who voted to close gave this specific reason:


  • "This question is missing context or other details: Please improve the question by providing additional context, which ideally includes your thoughts on the problem and any attempts you have made to solve it. This information helps others identify where you have difficulties and helps them write answers appropriate to your experience level." – amWhy, José Carlos Santos, Cesareo, Chinnapparaj R, user302797

If this question can be reworded to fit the rules in the help center, please edit the question.









  • 1




    Take $n=4$. Then $e^{n+1}approx148.4$, whereas $(1+n)^n=5^4=625$. So, you will never be able to prove that.
    – José Carlos Santos
    Nov 22 at 12:31










  • @JoséCarlosSantos My mistake, it should be $(1+n)^2$
    – Bartosz
    Nov 22 at 12:33










  • Where did you get stuck?
    – José Carlos Santos
    Nov 22 at 12:36










  • I end up with $e^{k+2} = e cdot e^{k+1} > e * (1+k)^2 > ... $
    – Bartosz
    Nov 22 at 12:42










  • It is enough to show that $f(x)=(x+1)^2 e^{-x}>0$ is bounded by $e$ on $mathbb{R}^+$. On the other hand $f'(x)$ only vanishes at $x=pm 1$, hence $f(x)leq f(1) = frac{4}{e} < e$.
    – Jack D'Aurizio
    Nov 22 at 21:02















up vote
0
down vote

favorite












I'm trying to prove by induction, but I'm getting stuck in the middle.



I'd use some help.










share|cite|improve this question













closed as off-topic by amWhy, José Carlos Santos, Cesareo, Chinnapparaj R, user302797 Nov 23 at 3:09


This question appears to be off-topic. The users who voted to close gave this specific reason:


  • "This question is missing context or other details: Please improve the question by providing additional context, which ideally includes your thoughts on the problem and any attempts you have made to solve it. This information helps others identify where you have difficulties and helps them write answers appropriate to your experience level." – amWhy, José Carlos Santos, Cesareo, Chinnapparaj R, user302797

If this question can be reworded to fit the rules in the help center, please edit the question.









  • 1




    Take $n=4$. Then $e^{n+1}approx148.4$, whereas $(1+n)^n=5^4=625$. So, you will never be able to prove that.
    – José Carlos Santos
    Nov 22 at 12:31










  • @JoséCarlosSantos My mistake, it should be $(1+n)^2$
    – Bartosz
    Nov 22 at 12:33










  • Where did you get stuck?
    – José Carlos Santos
    Nov 22 at 12:36










  • I end up with $e^{k+2} = e cdot e^{k+1} > e * (1+k)^2 > ... $
    – Bartosz
    Nov 22 at 12:42










  • It is enough to show that $f(x)=(x+1)^2 e^{-x}>0$ is bounded by $e$ on $mathbb{R}^+$. On the other hand $f'(x)$ only vanishes at $x=pm 1$, hence $f(x)leq f(1) = frac{4}{e} < e$.
    – Jack D'Aurizio
    Nov 22 at 21:02













up vote
0
down vote

favorite









up vote
0
down vote

favorite











I'm trying to prove by induction, but I'm getting stuck in the middle.



I'd use some help.










share|cite|improve this question













I'm trying to prove by induction, but I'm getting stuck in the middle.



I'd use some help.







calculus






share|cite|improve this question













share|cite|improve this question











share|cite|improve this question




share|cite|improve this question










asked Nov 22 at 12:29









Bartosz

315




315




closed as off-topic by amWhy, José Carlos Santos, Cesareo, Chinnapparaj R, user302797 Nov 23 at 3:09


This question appears to be off-topic. The users who voted to close gave this specific reason:


  • "This question is missing context or other details: Please improve the question by providing additional context, which ideally includes your thoughts on the problem and any attempts you have made to solve it. This information helps others identify where you have difficulties and helps them write answers appropriate to your experience level." – amWhy, José Carlos Santos, Cesareo, Chinnapparaj R, user302797

If this question can be reworded to fit the rules in the help center, please edit the question.




closed as off-topic by amWhy, José Carlos Santos, Cesareo, Chinnapparaj R, user302797 Nov 23 at 3:09


This question appears to be off-topic. The users who voted to close gave this specific reason:


  • "This question is missing context or other details: Please improve the question by providing additional context, which ideally includes your thoughts on the problem and any attempts you have made to solve it. This information helps others identify where you have difficulties and helps them write answers appropriate to your experience level." – amWhy, José Carlos Santos, Cesareo, Chinnapparaj R, user302797

If this question can be reworded to fit the rules in the help center, please edit the question.








  • 1




    Take $n=4$. Then $e^{n+1}approx148.4$, whereas $(1+n)^n=5^4=625$. So, you will never be able to prove that.
    – José Carlos Santos
    Nov 22 at 12:31










  • @JoséCarlosSantos My mistake, it should be $(1+n)^2$
    – Bartosz
    Nov 22 at 12:33










  • Where did you get stuck?
    – José Carlos Santos
    Nov 22 at 12:36










  • I end up with $e^{k+2} = e cdot e^{k+1} > e * (1+k)^2 > ... $
    – Bartosz
    Nov 22 at 12:42










  • It is enough to show that $f(x)=(x+1)^2 e^{-x}>0$ is bounded by $e$ on $mathbb{R}^+$. On the other hand $f'(x)$ only vanishes at $x=pm 1$, hence $f(x)leq f(1) = frac{4}{e} < e$.
    – Jack D'Aurizio
    Nov 22 at 21:02














  • 1




    Take $n=4$. Then $e^{n+1}approx148.4$, whereas $(1+n)^n=5^4=625$. So, you will never be able to prove that.
    – José Carlos Santos
    Nov 22 at 12:31










  • @JoséCarlosSantos My mistake, it should be $(1+n)^2$
    – Bartosz
    Nov 22 at 12:33










  • Where did you get stuck?
    – José Carlos Santos
    Nov 22 at 12:36










  • I end up with $e^{k+2} = e cdot e^{k+1} > e * (1+k)^2 > ... $
    – Bartosz
    Nov 22 at 12:42










  • It is enough to show that $f(x)=(x+1)^2 e^{-x}>0$ is bounded by $e$ on $mathbb{R}^+$. On the other hand $f'(x)$ only vanishes at $x=pm 1$, hence $f(x)leq f(1) = frac{4}{e} < e$.
    – Jack D'Aurizio
    Nov 22 at 21:02








1




1




Take $n=4$. Then $e^{n+1}approx148.4$, whereas $(1+n)^n=5^4=625$. So, you will never be able to prove that.
– José Carlos Santos
Nov 22 at 12:31




Take $n=4$. Then $e^{n+1}approx148.4$, whereas $(1+n)^n=5^4=625$. So, you will never be able to prove that.
– José Carlos Santos
Nov 22 at 12:31












@JoséCarlosSantos My mistake, it should be $(1+n)^2$
– Bartosz
Nov 22 at 12:33




@JoséCarlosSantos My mistake, it should be $(1+n)^2$
– Bartosz
Nov 22 at 12:33












Where did you get stuck?
– José Carlos Santos
Nov 22 at 12:36




Where did you get stuck?
– José Carlos Santos
Nov 22 at 12:36












I end up with $e^{k+2} = e cdot e^{k+1} > e * (1+k)^2 > ... $
– Bartosz
Nov 22 at 12:42




I end up with $e^{k+2} = e cdot e^{k+1} > e * (1+k)^2 > ... $
– Bartosz
Nov 22 at 12:42












It is enough to show that $f(x)=(x+1)^2 e^{-x}>0$ is bounded by $e$ on $mathbb{R}^+$. On the other hand $f'(x)$ only vanishes at $x=pm 1$, hence $f(x)leq f(1) = frac{4}{e} < e$.
– Jack D'Aurizio
Nov 22 at 21:02




It is enough to show that $f(x)=(x+1)^2 e^{-x}>0$ is bounded by $e$ on $mathbb{R}^+$. On the other hand $f'(x)$ only vanishes at $x=pm 1$, hence $f(x)leq f(1) = frac{4}{e} < e$.
– Jack D'Aurizio
Nov 22 at 21:02










3 Answers
3






active

oldest

votes

















up vote
3
down vote













Let consider



$$e^n>n^2$$



and we have




  • base case: $n=1$


  • induction step



$$e^{n+1}=ecdot e^n>ecdot n^2stackrel{?}>(n+1)^2$$






share|cite|improve this answer




























    up vote
    1
    down vote













    You can use the Taylor expansion of $e^{n+1}$.
    begin{align}
    e^{n+1}&=sum_{k=0}^inftyfrac{(n+1)^k}{k!}=1+1+n+frac{(n+1)^2}{2!}
    +frac{(n+1)^3}{3!}+O((n+1)^4)\
    &=1+n+1+frac{(n+1)^2}{2}+n^2/2+...=2+n+n^2+n+1/2+...\
    &=n^2+2n+1+3/2+...>(n+1)^2
    end{align}






    share|cite|improve this answer






























      up vote
      0
      down vote













      $n=1$ is ok.



      Our assumption: $e^n>n^2$.



      For $n>2ln(n)$ we must show that $n+1>2ln(n+1)$. $(n>1)$
      $$n+1>2ln(n)+1 stackrel{?}>2ln(n+1)$$
      $$1stackrel{?}>2ln(1+frac{1}{n})$$
      $$e>(1+frac{1}{n})^2 text{Because maximum of } (1+frac{1}{n})^2=2.25 text{Big idea is there} n>1 . $$






      share|cite|improve this answer




























        3 Answers
        3






        active

        oldest

        votes








        3 Answers
        3






        active

        oldest

        votes









        active

        oldest

        votes






        active

        oldest

        votes








        up vote
        3
        down vote













        Let consider



        $$e^n>n^2$$



        and we have




        • base case: $n=1$


        • induction step



        $$e^{n+1}=ecdot e^n>ecdot n^2stackrel{?}>(n+1)^2$$






        share|cite|improve this answer

























          up vote
          3
          down vote













          Let consider



          $$e^n>n^2$$



          and we have




          • base case: $n=1$


          • induction step



          $$e^{n+1}=ecdot e^n>ecdot n^2stackrel{?}>(n+1)^2$$






          share|cite|improve this answer























            up vote
            3
            down vote










            up vote
            3
            down vote









            Let consider



            $$e^n>n^2$$



            and we have




            • base case: $n=1$


            • induction step



            $$e^{n+1}=ecdot e^n>ecdot n^2stackrel{?}>(n+1)^2$$






            share|cite|improve this answer












            Let consider



            $$e^n>n^2$$



            and we have




            • base case: $n=1$


            • induction step



            $$e^{n+1}=ecdot e^n>ecdot n^2stackrel{?}>(n+1)^2$$







            share|cite|improve this answer












            share|cite|improve this answer



            share|cite|improve this answer










            answered Nov 22 at 12:37









            gimusi

            92.5k94495




            92.5k94495






















                up vote
                1
                down vote













                You can use the Taylor expansion of $e^{n+1}$.
                begin{align}
                e^{n+1}&=sum_{k=0}^inftyfrac{(n+1)^k}{k!}=1+1+n+frac{(n+1)^2}{2!}
                +frac{(n+1)^3}{3!}+O((n+1)^4)\
                &=1+n+1+frac{(n+1)^2}{2}+n^2/2+...=2+n+n^2+n+1/2+...\
                &=n^2+2n+1+3/2+...>(n+1)^2
                end{align}






                share|cite|improve this answer



























                  up vote
                  1
                  down vote













                  You can use the Taylor expansion of $e^{n+1}$.
                  begin{align}
                  e^{n+1}&=sum_{k=0}^inftyfrac{(n+1)^k}{k!}=1+1+n+frac{(n+1)^2}{2!}
                  +frac{(n+1)^3}{3!}+O((n+1)^4)\
                  &=1+n+1+frac{(n+1)^2}{2}+n^2/2+...=2+n+n^2+n+1/2+...\
                  &=n^2+2n+1+3/2+...>(n+1)^2
                  end{align}






                  share|cite|improve this answer

























                    up vote
                    1
                    down vote










                    up vote
                    1
                    down vote









                    You can use the Taylor expansion of $e^{n+1}$.
                    begin{align}
                    e^{n+1}&=sum_{k=0}^inftyfrac{(n+1)^k}{k!}=1+1+n+frac{(n+1)^2}{2!}
                    +frac{(n+1)^3}{3!}+O((n+1)^4)\
                    &=1+n+1+frac{(n+1)^2}{2}+n^2/2+...=2+n+n^2+n+1/2+...\
                    &=n^2+2n+1+3/2+...>(n+1)^2
                    end{align}






                    share|cite|improve this answer














                    You can use the Taylor expansion of $e^{n+1}$.
                    begin{align}
                    e^{n+1}&=sum_{k=0}^inftyfrac{(n+1)^k}{k!}=1+1+n+frac{(n+1)^2}{2!}
                    +frac{(n+1)^3}{3!}+O((n+1)^4)\
                    &=1+n+1+frac{(n+1)^2}{2}+n^2/2+...=2+n+n^2+n+1/2+...\
                    &=n^2+2n+1+3/2+...>(n+1)^2
                    end{align}







                    share|cite|improve this answer














                    share|cite|improve this answer



                    share|cite|improve this answer








                    edited Nov 22 at 13:24

























                    answered Nov 22 at 12:39









                    minmax

                    48518




                    48518






















                        up vote
                        0
                        down vote













                        $n=1$ is ok.



                        Our assumption: $e^n>n^2$.



                        For $n>2ln(n)$ we must show that $n+1>2ln(n+1)$. $(n>1)$
                        $$n+1>2ln(n)+1 stackrel{?}>2ln(n+1)$$
                        $$1stackrel{?}>2ln(1+frac{1}{n})$$
                        $$e>(1+frac{1}{n})^2 text{Because maximum of } (1+frac{1}{n})^2=2.25 text{Big idea is there} n>1 . $$






                        share|cite|improve this answer

























                          up vote
                          0
                          down vote













                          $n=1$ is ok.



                          Our assumption: $e^n>n^2$.



                          For $n>2ln(n)$ we must show that $n+1>2ln(n+1)$. $(n>1)$
                          $$n+1>2ln(n)+1 stackrel{?}>2ln(n+1)$$
                          $$1stackrel{?}>2ln(1+frac{1}{n})$$
                          $$e>(1+frac{1}{n})^2 text{Because maximum of } (1+frac{1}{n})^2=2.25 text{Big idea is there} n>1 . $$






                          share|cite|improve this answer























                            up vote
                            0
                            down vote










                            up vote
                            0
                            down vote









                            $n=1$ is ok.



                            Our assumption: $e^n>n^2$.



                            For $n>2ln(n)$ we must show that $n+1>2ln(n+1)$. $(n>1)$
                            $$n+1>2ln(n)+1 stackrel{?}>2ln(n+1)$$
                            $$1stackrel{?}>2ln(1+frac{1}{n})$$
                            $$e>(1+frac{1}{n})^2 text{Because maximum of } (1+frac{1}{n})^2=2.25 text{Big idea is there} n>1 . $$






                            share|cite|improve this answer












                            $n=1$ is ok.



                            Our assumption: $e^n>n^2$.



                            For $n>2ln(n)$ we must show that $n+1>2ln(n+1)$. $(n>1)$
                            $$n+1>2ln(n)+1 stackrel{?}>2ln(n+1)$$
                            $$1stackrel{?}>2ln(1+frac{1}{n})$$
                            $$e>(1+frac{1}{n})^2 text{Because maximum of } (1+frac{1}{n})^2=2.25 text{Big idea is there} n>1 . $$







                            share|cite|improve this answer












                            share|cite|improve this answer



                            share|cite|improve this answer










                            answered Nov 22 at 13:09









                            1ENİGMA1

                            960316




                            960316















                                Popular posts from this blog

                                Quarter-circle Tiles

                                build a pushdown automaton that recognizes the reverse language of a given pushdown automaton?

                                Mont Emei